Đến nội dung

Zz Isaac Newton Zz

Zz Isaac Newton Zz

Đăng ký: 27-06-2016
Offline Đăng nhập: 03-04-2024 - 08:51
****-

#696050 Đề thi chọn đội tuyển Học sinh giỏi môn Toán tỉnh Thanh Hóa năm 2017 - 2018.

Gửi bởi Zz Isaac Newton Zz trong 04-11-2017 - 14:16

Đề thi chọn đội tuyển Học sinh giỏi môn Toán tỉnh Thanh Hóa năm 2017 - 2018

Bài 1. Cho dãy số: $a_{0}, a_{1}, a_{2}, ...$ thỏa mãn: $a_{m+n}+a_{m-n}=\frac{1}{2}\left ( a_{2m}+a_{2n} \right ),$ với mọi số nguyên không âm $m, n$ và $m\geq n.$ Nếu $a_{1}=1,$ hãy xác định: $a_{2017}.$ 

Bài 2. Tìm tất cả các hàm số $f:\mathbb{R}\rightarrow \mathbb{R}$ thỏa mãn: $f(n^{2})=f(n+m).f(n-m)+m^{2}, \forall m, n\in \mathbb{R}.$

Bài 3. Tam giác $ABC$ nhọn có $H$ là trực tâm và $P$ là điểm di động bên trong tam giác sao cho $\widehat{BPC}=\widehat{BHC}.$ Đường thẳng qua $B$ và vuông góc với $AB$ cắt $PC$ tại $M,$ đường thẳng qua $C$ và vuông góc với $AC$ cắt $PB$ tại $N.$ Chứng minh rằng: trung điểm $I$ của $MN$ luôn thuộc một đường thẳng cố định.

Bài 4. Tìm tất cả các đa thức $P(x)$ có các hệ số nguyên thỏa mãn $P(2017)=1,$  $3^{n}-1$ chia hết cho $P(n)$ với mọi số nguyên dương $n.$

Bài 5. Chứng minh rằng: $\sum_{k=0}^{n}2^{k}C_{n}^{k}C_{n-k}^{\left [ \frac{n-k}{2} \right ]}=C_{2n+1}^{n}.$

*Đề thi có tham khảo ở link sau: http://olympictoanho...-2017-2018.html

 




#695773 Nếu $p$ là số nguyên tố dạng $6k+1$ thì tồn tại số nguyên...

Gửi bởi Zz Isaac Newton Zz trong 29-10-2017 - 16:35

1. Nếu $p$ là số nguyên tố dạng $6k+1$ thì tồn tại số nguyên $x$ sao cho $x^2+3 \vdots p$
2. Nếu $p$ là số nguyên tố dạng $6k+5$ thì không tồn tại số nguyên $x$ sao cho $x^2+3 \vdots p$
Thầy mình bảo giải 2 bài này bằng phương trình đồng dư.

1. Nếu $p$ là số nguyên tố có dạng $6k+1$ thì $p\equiv 1($$mod$ $6)$$\Rightarrow$ $p\equiv 1($$mod$ $3),$ vậy bài toán đã trở về chứng minh rằng: nếu $p$ là số nguyên tố có dạng $3k+1$ thì tồn tại số nguyên $x$ sao cho $x^{2}+3$ $\vdots$ $p.$ Mà dễ thấy $p\neq 3$ nên

Bây giờ ta sẽ chứng minh một điều ngược lại là: Nếu $p$ là số nguyên tố có dạng $3k+2$ thì không tồn tại số nguyên $x$ sao cho $x^{2}+3$ $\vdots$ $p.$

Giả sử tồn tại số nguyên $x$ sao cho $x^{2}+3$$\vdots$ $p$ hay $x^{2}+3\equiv0$$($$mod$$p)$$\Rightarrow x^{2}\equiv -3$$($$mod$ $p)$ $\Rightarrow \left ( \frac{-3}{p} \right )=1.$

Theo định lý tiêu chuẩn $Euler$ ta có: $1=\left ( \frac{-3}{p} \right )=\left ( \frac{-1}{p} \right )\left ( \frac{3}{p} \right )=\left ( -1 \right )^{\frac{p-1}{2}}\left ( \frac{3}{p} \right ).$

Theo luật tương hỗ $Gauss$ ta có: $\left ( \frac{3}{p} \right )\left ( \frac{p}{3} \right )=\left ( -1 \right )^{\frac{(3-1)(p-1)}{4}}=\left ( -1 \right )^{\frac{p-1}{2}}\Rightarrow \left ( \frac{3}{p} \right )=\left ( -1 \right )^{\frac{p-1}{2}}\left ( \frac{p}{3} \right ).$

Từ đây ta suy ra: $1=\left ( \frac{-3}{p} \right )=\left ( -1 \right )^{\frac{p-1}{2}}\left ( -1 \right )^{\frac{p-1}{2}}\left ( \frac{p}{3} \right )=\left ( -1 \right )^{p-1}\left ( \frac{p}{3} \right )$      ($*$)

Mà  $p$ là số nguyên tố có dạng $3k+2$ nên $p\equiv 2($$mod$ $3)$$\Rightarrow \left ( \frac{p}{3} \right )=\left ( \frac{2}{3} \right )=\left ( -1 \right )^{\frac{3^{2}-1}{8}}=-1,$ từ đây thay vào ($*$) ta được: $1=\left ( -1 \right )^{p-1}\left ( -1 \right )=\left ( -1 \right )^{p}=-1,$ điều này vô lý nên suy ra điều phải chứng minh. Hay nếu $p$ là số nguyên tố có dạng $3k+1$ thì tồn tại số nguyên $x$ sao cho $x^{2}+3$ $\vdots$ $p.$ Vậy nếu $p$ là số nguyên tố có dạng $6k+1$ thì tồn tại số nguyên $x$ sao cho $x^{2}+3$ $\vdots$ $p.$

2. Nếu $p$ là số nguyên tố có dạng $6k+5$ thì $p\equiv 2($$mod$ $3),$ mà theo câu 1. thì không tồn tại số nguyên $x$ sao cho $x^{2}+3$ $\vdots$ $p.$ Vậy từ đây suy ra, nếu $p$ là số nguyên tố có dạng $6k+5$ thì không tồn tại số nguyên $x$ sao cho $x^{2}+3$ $\vdots$ $p.$




#695705 $p^n+q^n=r^2$

Gửi bởi Zz Isaac Newton Zz trong 28-10-2017 - 16:24

Cho $p,q,r$ là các số nguyên tố và $n$ là số nguyên dương thỏa mãn

                                      $p^{n}+q^{n}=r^{2}$

Chứng minh: $n=1$.

Trước hết ta có thể giả sử $q=2$

* Nếu $n$ là số nguyên dương lẻ thì ta có: $p^{n}+2^{n}=\left ( p+2 \right )\left ( \frac{p^{n}+2^{n}}{p+2} \right )=r^{2},$ mà do $r$ là số nguyên tố nên ta phải có: $p+2=\frac{p^{n}+2^{n}}{p+2}=r.$

Nếu $n$ là số lẻ và $n\geq 3$ thì ta có: $\frac{p^{n}+2^{n}}{p+2}> p+2, $ từ đây ta dẫn đến một điều vô lý. Do đó, ta phải có: $n=1.$

* Nếu $n$ là số chẵn, đặt $n=2k, k\in \mathbb{Z}^{+}$ thì từ đây ta có: $\left ( p^{k} \right )^{2}+\left ( 2^{k} \right )^{2}=r^{2},$ mà dễ thấy $p, r$ phải phân biệt nên đây là bộ ba $Phythagore$ nên tồn tại $x, y: \left ( x, y \right )=1$ và $x, y$ khác tính chẵn lẻ thỏa mãn: $\left\{\begin{matrix} p^{k}=2xy & & \\ 2^{k}=x^{2}-y^{2} & & \end{matrix}\right.$ hoặc $\left\{\begin{matrix} 2^{k}=2xy & & \\ p^{k}=x^{2}-y^{2} & & \end{matrix}\right.$ Mà $p$ là số nguyên tố nên trường hợp này không xảy ra.

Vậy ta phải có: $n=1.$




#695674 Chứng minh rằng số các toàn ánh bằng $\sum_{k=0}^{n-...

Gửi bởi Zz Isaac Newton Zz trong 27-10-2017 - 21:02

Cho $m\geq n$ là hai số nguyên dương. Chứng minh rằng số các toàn ánh $f:\left \{ 1, 2, 3, ..., m \right \}\rightarrow \left \{ 1, 2, 3, ..., n \right \}$ bằng $\sum_{k=0}^{n-1}(-1)^{k}\binom{n}{k}(n-k)^{m}.$




#695482 $\lim b_n$

Gửi bởi Zz Isaac Newton Zz trong 25-10-2017 - 20:30

Cho $(x_n):x_1=a(a \geq 2);x_n=(x_{n-1}^2)-2$ và $b_n= \frac{1}{x_1}+ \frac{1}{x_1x_2}+..+ \frac{1}{x_1x_2...x_n}.$

Tìm $\lim b_n.$

Với mọi $n=1, 2, 3, ...$ ta có: $x_{n}=x_{n-1}^{2}-2\Leftrightarrow x_{n+1}=x_{n}^{2}-2\Leftrightarrow x_{n+1}^{2}-4=\left ( x_{n}^{2}-2 \right )^{2}-4=x_{n}^{4}-4x_{n}^{2}=x_{n}^{2}\left ( x_{n}^{2}-2 \right )=x_{n}^{2}x_{n-1}^{2}\left ( x_{n-1}^{2}-4 \right )=...=x_{n}^{2}x_{n-1}^{2}...x_{2}^{2}x_{1}^{2}\left ( x_{1}^{2}-4 \right )=\left ( x_{1}x_{_{2}}...x_{n} \right )^{2}.\left ( a^{2}-4 \right )$ $\Leftrightarrow \left ( \frac{x_{n+1}}{x_{1}x_{2}...x_{n}} \right )^{2}=a^{2}-4+\frac{4}{\left ( x_{1}x_{2}...x_{n} \right )^{2}}.$

Mặt khác vì: $x_{1}=a\geq 2\Rightarrow x_{n}\geq 2, \forall n=1, 2, 3, ....$ Do đó: $x_{1}x_{2}....x_{_{n}}> 2^{n}, \forall n\in \mathbb{N}^{*}.$ Bởi vậy: $0< \frac{4}{\left ( x_{1}x_{_{2}}...x_{n} \right )^{2}}< \frac{4}{2^{2n}}, \forall n=1, 2, 3, ...$ nên theo nguyên lý kẹp ta có: $\lim_{n\rightarrow +\infty }\frac{4}{\left ( x_{1}x_{2}...x_{n} \right )^{2}}=0,$ từ đây suy ra: $\lim_{n\rightarrow +\infty }\frac{x_{n+1}}{x_{1}x_{2}...x_{n}}=\sqrt{a^{2}-4}.$      (1)

Mà ta lại có: $\frac{1}{x_{1}x_{_{2}}...x_{n}}=\frac{2}{2x_{1}x_{2}...x_{n}}=\frac{x_{n}^{2}-x_{n+1}}{2x_{1}x_{2}...x_{_{n}}}=\frac{1}{2}\left ( \frac{x_{n}}{x_{1}x_{2}...x_{n-1}}-\frac{x_{n+1}}{x_{1}x_{2}...x_{n}} \right ),$ từ đây ta có: $b_{n}=\frac{1}{x_{1}}+\frac{1}{x_{1}x_{2}}+...+\frac{1}{x_{1}x_{2}...x_{n}}=\frac{1}{x_{1}}+\left ( \frac{1}{x_{1}x_{2}}+...+\frac{1}{x_{1}x_{2}...x_{n}} \right )=\frac{1}{a}+\frac{1}{2}\left [ \left ( \frac{x_{2}}{x_{1}}-\frac{x_{3}}{x_{1}x_{2}} \right )+...+\left ( \frac{x_{n}}{x_{1}x_{_{2}}...x_{n-1}}-\frac{x_{n+1}}{x_{1}x_{2}...x_{n}} \right ) \right ]=\frac{1}{a}+\frac{x_{2}}{2x_{_{1}}}-\frac{x_{n+1}}{x_{1}x_{2}...x_{_{n}}}=\frac{1}{a}+\frac{a^{2}-2}{2a}-\frac{x_{n+1}}{2x_{1}x_{2}...x_{n}}.$

Từ đây sử dụng (1) ta có: $\lim_{n\rightarrow +\infty }b_{_{n}}=\lim_{n\rightarrow +\infty }\left ( \frac{1}{x_{1}}+\frac{1}{x_{1}x_{2}}+...+\frac{1}{x_{1}x_{2}...x_{n}}\right )=\frac{1}{a}+\frac{a^{^{2}}-2}{2a}-\frac{\sqrt{a^{2}-4}}{2}=\frac{a-\sqrt{a^{2}-4}}{2}.$




#695007 Định lý Kazandzidis.

Gửi bởi Zz Isaac Newton Zz trong 17-10-2017 - 21:39

Cho số nguyên tố $p> 3$ và hai số nguyên dương $n, k.$ Chứng minh rằng: $\binom{pn}{pk}\equiv \binom{n}{k}$ $(mod$ $Q ),$ với $Q=p^{m}$ và trong đó: $m=3+v_{p}\left ( nk(n-k)\binom{n}{k} \right ).$




#694777 $\frac{a^{2}}{a+2b^{2}}+\frac{b^{2}}{b+2c^{2}}+\frac{c^{2...

Gửi bởi Zz Isaac Newton Zz trong 14-10-2017 - 21:12

Dùng kĩ thuật $Cauchy$ ngược dấu nha bạn...




#694600 Chứng minh rằng với mọi số nguyên $k\geq 2$ thì phương trình k...

Gửi bởi Zz Isaac Newton Zz trong 11-10-2017 - 20:43

Chứng minh rằng với mọi số nguyên $k\geq 2$ thì phương trình: $\frac{1}{10^{n}}=\frac{1}{n_{1}!}+\frac{1}{n_{2}!}+ ...+\frac{1}{n_{k}!}$ không có nghiệm nguyên thỏa mãn $1\leq n_{1}< n_{2}< ...< n_{k}.$




#693327 $-3.\frac{2^{p-1}-1}{p}$

Gửi bởi Zz Isaac Newton Zz trong 18-09-2017 - 22:00

Cho $p$ là một số nguyên tố lẻ. Chứng minh rằng: $-3.\frac{2^{p-1}-1}{p}\equiv \sum_{k=1}^{\begin{bmatrix} \frac{p}{4} \end{bmatrix}}\frac{1}{k}$ $(mod$ $p).$




#692478 $f(m+n)+f(mn-1)=f(m)f(n)+2$

Gửi bởi Zz Isaac Newton Zz trong 06-09-2017 - 17:10

Tìm tất cả các hàm số$\mathbb{Z}\rightarrow \mathbb{Z}$ thỏa mãn:$f(m+n)+f(mn-1)=f(m)f(n)+2$ (1)

Nếu $f(n)=C,$ với $C$ là hằng số thì thế vào (1) ta được: $2C=C^{2}+2,$ phương trình này vô nghiệm, vậy suy ra $f$ không phải là hàm hằng.

Từ (1), thế $m=0$ ta được: $f(n)+f(-1)=f(0).f(n)+2, \forall n\in \mathbb{Z}.$

$\Leftrightarrow \left [ f(0)-1 \right ].f(n)=f(-1)-2, \forall n\in \mathbb{Z}$ (2)

Nếu $f(0)-1\neq 0$ thì từ (2) suy ra $f$ là hàm hằng, vô lý.

Vậy $f(0)=1$ và $f(-1)=2.$

Từ (1) thế $m=-1$ ta được: $f(n-1)+f(-n-1)=2f(n)+2, \forall n\in \mathbb{Z}$ (3).

Từ (3) thế $n$ bởi $-n$ ta được: $f(-n-1)+f(n-1)=2f(-n)+2, \forall n\in \mathbb{Z}$ (4).

Từ (3) và (4) suy ra: $f(n)=f(-n), \forall n\in \mathbb{Z}$ và (3) trở thành:

$f(n-1)+f(n+1)=2f(n)+2, \forall n\in \mathbb{Z}$ (5).

Xét dãy số $\left ( x_{n} \right )_{n=0}^{+\infty }$ như sau: $x_{n}=f(n), \forall n=0, 1, 2, ...$

Từ (5) ta có: $x_{n+1}+x_{n-1}=2x_{n}+2, \forall n\in \mathbb{N}.$

Giải phương trình này ta được: $x_{n}=n^{2}+1\Rightarrow f(n)=n^{2}+1, \forall n\in \mathbb{N}.$

Do $f$ là hàm chẵn trên $\mathbb{Z}$ nên ta suy ra: $f(n)=n^{2}+1, \forall n\in \mathbb{Z}.$

Thử lại thấy thỏa mãn.




#691624 Chứng minh rằng: $\sum \frac{1}{1+\sqrt...

Gửi bởi Zz Isaac Newton Zz trong 26-08-2017 - 21:28

 

Bài này là đề thi học sinh giỏi hay đề anh chế vậy.Căng não cả buổi trời mới ra.

 

Trước hết , để làm được bài này ta có các bổ đề sau:

 

(1) $(ab)^{2}+(bc)^{2}+(ca)^{2}\geq abc(a+b+c)$

 

(2) $\frac{1}{a+b+c}\leq \frac{1}{9}(\frac{1}{a}+\frac{1}{b}+\frac{1}{c})$

 

(3) $\frac{1}{a+b+c+d}\leq \frac{1}{16}\sum \frac{1}{a}$

 

(4) $\sum \frac{1}{a^{3}+b^{3}+abc}\leq \frac{1}{abc}$

 

Những bđt này các bạn có thể tìm cách chứng minh trên mạng.

 

Quay lại bài toán.

 

Từ gt => 

 

$(ab)^{2}+(bc)^{2}+(ca)^{2}=3(abc)^{2}$

Bằng cách sử dụng (1) , ta được:

$3(abc)^{2}\geq abc(a+b+c)$

=> $3abc\geq a+b+c$

Ta có: P = $\sum \frac{1}{1+\sqrt{(a+b)^{3}+abc}}\leq \frac{1}{16}(3+\sum \frac{9}{\sqrt{(a+b)^{3}+abc}})$ (theo (3))

Ta cần cm $\frac{1}{16}(3+\sum \frac{9}{\sqrt{(a+b)^{3}+abc}}) \leq \frac{3}{4}$

<=> $\sum \frac{1}{\sqrt{(a+b)^{3}+abc}}\leq 1$

<=> $(\sum \frac{1}{\sqrt{(a+b)^{3}+abc}})^{2}\leq 1$

Ta sẽ cm bất đẳng thức trên. Thật vậy, ta có:

$(\sum \frac{1}{\sqrt{(a+b)^{3}+abc}})^{2}\leq 3.\sum \frac{1}{(a+b)^{3}+abc}$ (Cauchy-Schwarz)

. $\sum \frac{1}{(a+b)^{3}+abc}=\sum \frac{1}{(a^{3}+b^{3}+abc)+3ab(a+b)}\leq \frac{1}{9}(\sum \frac{1}{a^{3}+b^{3}+abc}+\sum \frac{4}{3ab(a+b)})$(theo (2)) $\leq \frac{1}{9}(\frac{1}{abc}+\sum \frac{1}{3(a+b)}.(\frac{1}{a}+\frac{1}{b})^{2})$ (Theo (4) và bđt AM-GM) $\leq \frac{1}{9}(\frac{1}{abc}+\sum \frac{(a+b)^{2}}{3(a+b)(ab)^{2}})=\frac{1}{9}(\frac{1}{abc}+\sum \frac{(a+b)}{3(ab)^{2}})=\frac{1}{9}(\frac{1}{abc}+\sum \frac{1}{3ab}(\frac{1}{a}+\frac{1}{b}))=\frac{1}{9}(\frac{1}{abc}+\sum \frac{1}{a}(\frac{1}{3ab}+\frac{1}{3ca}))=\frac{1}{9}(\frac{1}{abc}+\sum \frac{1}{3abc}(\frac{b+c}{a}))=\frac{1}{9}(\frac{1}{abc}+\frac{1}{3abc}(\frac{b+c}{a}+\frac{c+a}{b}+\frac{a+b}{c}))=\frac{1}{9}(\frac{1}{abc}+\frac{1}{3abc}(\frac{b+c}{a}+\frac{c+a}{b}+\frac{a+b}{c}+3-3))=\frac{1}{9}(1+\frac{1}{3abc})=\frac{(a+b+c)}{27abc}(\frac{1}{a}+\frac{1}{b}+\frac{1}{c})=\frac{(a+b+c)}{27abc}(\frac{ab+bc+ca}{abc})\leq \frac{3abc}{27abc}\frac{\sqrt{3.((ab)^{2}+(bc)^{2}+(ca)^{2})}}{abc}=\frac{1}{9}\frac{\sqrt{3.3(abc)^{2}}}{abc}=\frac{1}{3}$

=> $(\sum \frac{1}{\sqrt{(a+b)^{3}+abc}})^{2}\leq 3.\sum \frac{1}{(a+b)^{3}+abc}\leq 3.\frac{1}{3}=1$

=>$(\sum \frac{1}{\sqrt{(a+b)^{3}+abc}})^{2}\leq 1$

=> Bđt được chứng minh. Dấu bằng xảy ra <=> a=b=c=1

=> Đpcm.

 

Bài này anh chỉ sưu tầm đề thôi em, anh làm như thế này, không biết có ổn không, em xem thử...

Ta có: $\frac{1}{a^{2}}+\frac{1}{b^{2}}+\frac{1}{c^{2}}=3\geq \frac{3}{\sqrt[3]{a^{2}b^{2}c^{2}}}\Leftrightarrow abc\geq 1.$

Từ đây: $\sum \frac{1}{1+\sqrt{(a+b)^{3}+abc}}\leq \sum \frac{1}{1+\sqrt{(a+b)^{3}+1}}=\sum \frac{\sqrt{(a+b)^{3}+1}-1}{(a+b)^{3}}=\sum \frac{\sqrt{(a+b+1)\left ( (a+b)^{2}-(a+b)+1 \right )}-1}{(a+b)^{3}}\leq \sum \frac{(a+b)+1+(a+b)^{2}-(a+b)+1-2}{2(a+b)^{3}}=\sum \frac{1}{2(a+b)}$$\leq \frac{1}{2}.\frac{1}{4}(\frac{1}{a}+\frac{1}{b}+\frac{1}{b}+\frac{1}{c}+\frac{1}{c}+\frac{1}{a})=\frac{1}{4}(\frac{1}{a}+\frac{1}{b}+\frac{1}{c})\leq \frac{1}{4}.\sqrt{3\left ( \frac{1}{a^{2}}+\frac{1}{b^{2}}+\frac{1}{c^{2}} \right )}=\frac{3}{4}.$




#690701 1,cho x,y,z>0 và x+y+z=$2\sqrt{2}$ tìm GTLN P=...

Gửi bởi Zz Isaac Newton Zz trong 16-08-2017 - 21:53

Bài 2 dùng bất đẳng thức $Minkowski$ và bất đẳng thức $Cauchy$ là xong.




#690463 SỐ HỌC QUA CÁC ĐỊNH LÝ VÀ BÀI TOÁN - Trần Nam Dũng

Gửi bởi Zz Isaac Newton Zz trong 13-08-2017 - 21:04

Đây nha bạn...

File gửi kèm




#689634 $f(xy)=max\left \{ f(x+y), f(x).f(y) \right \}$

Gửi bởi Zz Isaac Newton Zz trong 05-08-2017 - 16:13

 

$\mathcal{P}(x,1)\rightarrow f(x)\ge f(x+1),\ \forall x\ \ \ (1)$

$\mathcal{P}(x,0)\rightarrow f(0)=\max\left \{ f(x),f(x).f(0) \right \}\ \ \ (2)$

$\bullet\ \boxed{\text{TH1:}\ f(0)=0}$

$\mathcal{P}(x,-x)\rightarrow f(-x^2)=\max\left \{ 0,f(x).f(-x)\right \}\ \ (3)$

từ đây ta có

$f(x)\ge 0,\ \forall x\le 0\ \ \ (4)$

tới đây ta xét $3$ tập $\mathcal{A},\mathcal{B},\mathcal{C}$ là

$\left\{\begin{matrix}f(x)>0,\forall x \in \mathcal{A} \\f(x)<0,\forall x \in \mathcal{B} \\ f(x)=0,\forall x \in \mathcal{C} \end{matrix}\right.$

tới đây ta xem $a,b,c$ lần lượt là các phần tử thuộc các tập $\mathcal{A},\mathcal{B},\mathcal{C}$

ta giả sử $\left | \mathcal{C} \right |=1$ tức là tập $\mathcal{C}$ chỉ có duy nhất phần tử là $0$

$(4)\Rightarrow f(x)>0,\ \ \forall x<0$

mà từ $(2)$ ta có

$0=f(0)\ge f(x),\ \ \forall x$

từ 2 điều trên ta có mâu thuẫn do đó $\left | \mathcal{C} \right |>1$

$\mathcal{P}(x,c)\rightarrow f(xc)=\max \left \{ f(x+c),0\right \}$

do đó $f(xc)\ge 0,\ \forall x$ và ta có thể chọn $x$ cùng với $c\neq 0$ nên 

$f(x)\ge 0,\ \ \forall x\Rightarrow \mathcal{B}=\varnothing$

từ $(1)$ ta có $0=f(0)\ge f(1)\ge 0\rightarrow f(1)=0$

với $x\neq 0$ ta chọn

$\mathcal{P}\left ( x,\frac{1}{x} \right )\rightarrow 0=f\left ( x.\frac{1}{x} \right )=\max\left \{ f\left ( x+\frac{1}{x} \right ),f(x).f\left ( \frac{1}{x} \right ) \right \}$

từ đây ta có được $f\left ( x+\frac{1}{x}\right )=0$ tức là

$x+\frac{1}{x}\in \mathcal{C},\ \forall x\neq 0$

tới đây vì $x+\frac{1}{x}\ge 2$ nên $\forall y:\left | y \right |\ge 2\rightarrow \exists x_0:y=x_0+\frac{1}{x_0}$ do đó

$\forall x:\left | x \right |\ge 2\rightarrow x\in \mathcal{C}$

từ đây với giả thiết đầu bài ta cố định $x$ và chọn $y$ đủ lớn sao cho $\left\{\begin{matrix} \left | x+y \right |\ge 2\\\left | xy \right |\ge 2 \end{matrix}\right.$ nên 

$f(xy)=\max\left \{ f(x+y),f(x).f(y)\right \}=\max\left \{ 0,0 \right \}=0$

từ đây với mỗi $x$  cố định ta đều có thể chọn $y$ mà $f(xy)=0$ nên ta có thể suy ra 

$f(x)=0,\ \ \forall x$

$\bullet\ \boxed{\text{TH2:}\ f(0)\neq 0}$

với trường hợp này từ $(2)$ ta suy ra tập giá trị của hàm $f$ là $1$ hoặc $f(0)$

nếu $f(0)=1$ thì không còn gì để nói nên ta xét với $f(0)\neq 1$

mà $\mathcal{P}(0,0)\rightarrow f(0)\ge f(0)^2\Rightarrow f(0)\le 1$ nên $f(0)<1$

tới đây ý tưởng cũng như trên ta xét hai tập $\mathcal{D},\mathcal{E}$ là

$\left\{\begin{matrix} f(x)=1,\forall x\in \mathcal{D}\\ f(x)=f(0),\forall x\in \mathcal{E} \end{matrix}\right.$

từ $(1)$ ta có $f(0)\ge f(1)$ mà ta thấy không còn giá trị nào của hàm $f$ bé hơn $f(0)$ nên $f(1)=f(0)$

tới đây thì tương tự như trên là

$\mathcal{P}\left ( x,\frac{1}{x} \right )\rightarrow f(0)=f\left ( x.\frac{1}{x} \right )=\max\left \{ f\left ( x+\frac{1}{x} \right ),f(x).f\left ( \frac{1}{x} \right ) \right \}$

từ đây ta có được

$x+\frac{1}{x}\in \mathcal{E},\ \forall x\neq 0$

tới đây phần còn lại giải quyết hoàn toàn tương tự như trên

và đáp số bài toán là $f(x)\equiv C,\ \ \forall C<1$

Spoiler

 

Cách làm của anh ảo diệu thật, em làm như thế này, không biết có ổn không, mong anh chỉ giáo,...

Trong (1) lấy $x=0$ ta được: $f(0)=max\left \{ f(y), f(0).f(y) \right \}, \forall y\in \mathbb{R}$ (2)

Từ (2) lấy $y=0$ ta được: $f(0)=max\left \{ f(0), f^{2}(0) \right \}$  $\Rightarrow f(0)\geq f^{2}(0)\Leftrightarrow 0\leq f(0)\leq 1.$

Giả sử $\exists y\in \mathbb{R}$ sao cho $f(y)< 0,$ khi đó ta có: $f(y)\leq f(0).f(y),$ từ đây kết hợp với (2) suy ra: $f(0)=f(0).f(y)\Leftrightarrow f(0).(f(y)-1)=0\Leftrightarrow f(0)=0.$

Vậy (2) trở thành $0=max\left \{ f(y), 0 \right \}, \forall y\in \mathbb{R}\Rightarrow f(y)\leq 0, \forall y\in \mathbb{R}.$

*Nếu $\exists y\in \mathbb{R}:f(y)< 0$ thì $f(x)\leq 0, \forall x\in \mathbb{R}.$ Do đó theo (1) suy ra: $f(x)=max\left \{ f(x+1), f(x).f(1) \right \}= f(x).f(1)\geq 0, \forall x\in \mathbb{R}.$

Do đó: $f(x)=0=f(0), \forall x\in \mathbb{R}.$

*Nếu $f(y)\geq 0, \forall y\in \mathbb{R}$ thì $f(y)\geq f(0).f(y), \forall y\in \mathbb{R}.$ Do đó (2) trở thành $f(y)=f(0), \forall y\in \mathbb{R}.$

Vậy từ đây cho ta: $f(x)=C, \forall x\in \mathbb{R}$ và $C$ là hằng số. Thay vào (1) ta được: $C=max\left \{ C, C^{2} \right \}\Rightarrow C\geq C^{2}\Leftrightarrow C\in \left [ 0, 1 \right ].$

Hàm số thỏa mãn yêu cầu đề bài là: $f(x)=C, \forall x\in \mathbb{R}$ và $C\in \left [ 0, 1 \right ]$ là hằng số.




#687067 Chứng minh rằng: $m\leq C_{n-1}^{k-1}.$

Gửi bởi Zz Isaac Newton Zz trong 09-07-2017 - 22:07

Cho tập $S$ có $n$ phần tử và $F=\left \{ A_{1}, A_{2}, ..., A_{m} \right \}$ là các tập con gồm $k$ phần tử của $S,$ $k\leq \frac{n}{2}$ sao cho $\forall i, j\in \left \{ 1, 2, ..., k \right \}; i\neq j$ thì $\begin{vmatrix} A_{i}\cap A_{j} \end{vmatrix} \geq 1.$ Chứng minh rằng: $m\leq C_{n-1}^{k-1}.$


  • JUV yêu thích